LSAT and Law School Admissions Forum

Get expert LSAT preparation and law school admissions advice from PowerScore Test Preparation.

User avatar
 Dave Killoran
PowerScore Staff
  • PowerScore Staff
  • Posts: 5852
  • Joined: Mar 25, 2011
|
#41162
Complete Question Explanation
(The complete setup for this game can be found here: lsat/viewtopic.php?t=11758)

The correct answer choice is (B)

As discussed during the setup, I and G are in the level 1 class. The only uncertainty is whether J or K is the third member of the class. Thus, there two possible combinations for level 1—I, G, J and I, G, K—and answer choice (B) is correct.
User avatar
 ccanno
  • Posts: 4
  • Joined: Mar 04, 2021
|
#84870
Hi I don't seem to understand how answer choice B is the correct answer. If the two lesson 1 combinations both involve Glen, how can he be an option for lesson 2?
 Rachael Wilkenfeld
PowerScore Staff
  • PowerScore Staff
  • Posts: 1358
  • Joined: Dec 15, 2011
|
#84888
Hi ccano,

For this question, they ask us to determine how many groups of students could be the correct group of students in Level 1. We don't really have to consider level 2 at all here.

Since I has to go before everyone, and G must be before everyone but I, we know that level 1 has to include both I and G. We don't know if J or K comes next. So we have two different possible groups here

1) I, G, and J
2) I, G, and K

We can't have anyone else in level 1, because all of the other students must be after either J or K. We also can't have more than three students in each level.

Hope that helps!

Get the most out of your LSAT Prep Plus subscription.

Analyze and track your performance with our Testing and Analytics Package.